Generally healthy back
Are you saying that a generally healthy back is not a sufficient condition? If so, then I am not ...
Nikki37 on December 21, 2019
  • June 2007 LSAT
  • SEC3
  • Q17
4
Replies
help
having a hard time with this question
Lucas on November 26, 2019
  • June 2007 LSAT
  • SEC3
  • Q10
2
Replies
Why is the answer E and not D?
I'm confused. Why is the answer E and not D?
Zariyah-Hodge on November 25, 2019
  • June 2007 LSAT
  • SEC3
  • Q12
9
Replies
How is answer choice B the correct answer
How is answer choice B the correct answer as I don't see anywhere in the premise that talks about...
Nikki37 on October 11, 2019
  • June 2007 LSAT
  • SEC3
  • Q16
1
Reply
How does A strengthen
A makes perfect sense if the question was a weaken question or would strengthen the argument if t...
mgraves5 on August 24, 2019
  • June 2007 LSAT
  • SEC3
  • Q19
2
Replies
Why is E incorrect?
Why would the inability for farmland to sustain anything else but animal farming relate to the mo...
kokolyia on August 9, 2019
  • June 2007 LSAT
  • SEC3
  • Q21
3
Replies
Why is B wrong
Can anyone provide a different way of explaining why B is wrong?
Meredith on August 8, 2019
  • June 2007 LSAT
  • SEC3
  • Q24
2
Replies
s/n condition diagram please
I answered with B. Can you explain please?
yoyo33 on August 1, 2019
  • June 2007 LSAT
  • SEC3
  • Q22
4
Replies
thrown for a loop
this one really is confusing me, as I don't see the connection between institutions effects on pe...
antigonus on June 19, 2019
  • June 2007 LSAT
  • SEC3
  • Q24
2
Replies
Flawed parallel reasoning
After watching the video explanation I'm still having trouble with understanding how one would ev...
tainadiaz on May 30, 2019
  • June 2007 LSAT
  • SEC3
  • Q20
2
Replies
Question #6 Explanation
Hello. I would like to know why the answer to #6 is B and not anything else?
JRO on May 24, 2019
  • June 2007 LSAT
  • SEC3
  • Q16
1
Reply
Why is E incorrect?
I chose E, which was incorrect. Is it because what nutritionists' have to say is irrelevant?
anam2020 on May 18, 2019
  • June 2007 LSAT
  • SEC3
  • Q21
3
Replies
Answer choices
It is recommendable to read the answer choices first? Or go straight to the paragraph?
Barbie231 on September 26, 2017
  • June 2007 LSAT
  • SEC3
  • Q12
3
Replies
Help
Could you explain why (C) is the answer? Thanks!
neoneon on November 21, 2015
  • June 2007 LSAT
  • SEC3
  • Q1
4
Replies